LSAT and Law School Admissions Forum

Get expert LSAT preparation and law school admissions advice from PowerScore Test Preparation.

 Administrator
PowerScore Staff
  • PowerScore Staff
  • Posts: 8919
  • Joined: Feb 02, 2011
|
#71266
Please post your questions below! Thank you!
 juandresmc
  • Posts: 21
  • Joined: Dec 12, 2019
|
#75363
Hello PS!

I correctly chose answer choice (E) but had a tough time eliminating answer choice (C). My reasoning for eliminating it was that the language was too extreme (“none ... has grown significantly”). Can someone please explain why (E) is correct and (C) is wrong?

Thank you very much.

Regards,

Andrés
 Jeremy Press
PowerScore Staff
  • PowerScore Staff
  • Posts: 1000
  • Joined: Jun 12, 2017
|
#75377
Hi Andres,

You're correct that answer choice C is too extreme an answer to follow from the information given in the stimulus. Both the firms and the economist are talking about growing "big enough to compete successfully with foreign rivals," an absolute size. So all we can know from the stimulus is that the firms haven't attained that absolute size and, according to the economist, protection won't help them reach that absolute size (since they didn't reach it within ten years).

But the stimulus gives us no information on how much any of the firms has grown in the last ten years, i.e. no information about the firms' change in size. They might have started out very small and become very big (a significant change in size), but still not become "big enough to compete successfully." We cannot know, and since we cannot know, answer choice C is not supported.

I hope this helps!

Jeremy
 juandresmc
  • Posts: 21
  • Joined: Dec 12, 2019
|
#75384
Hello Jeremy!

Thank you very much, now I get why (E) is the correct and (C) is wrong. Greetings from Guatemala!

Regards,

Andrés
 LSAT-Learner
  • Posts: 4
  • Joined: May 15, 2020
|
#75830
Hello!

Can you please explain why D is incorrect?
 Jeremy Press
PowerScore Staff
  • PowerScore Staff
  • Posts: 1000
  • Joined: Jun 12, 2017
|
#75873
Hi LSAT Learner,

There are a couple things wrong with answer choice D here. First, while we know from the stimulus that there is a problem with the firms' "competing successfully" with foreign machinery firms, we do not know whether their inability to compete successfully necessarily means they will "go out of business." Maybe the domestic firms will simply limp along being less successful (in relation to the foreign ones), but won't entirely go out of business. So that new terminology in answer choice D is problematic.

Second, and more importantly, answer choice D doesn't fit the logical relationship the stimulus establishes between protection and ability to compete successfully. The stimulus establishes that protection from competition will never be sufficient to allow machinery firms to compete successfully, because, if it were, that would've happened already within the ten years they were protected. What answer choice D says is that protection is necessary ("unless" being a necessary condition indicator) for the firms to survive. That's a different logical statement than to say protection is not sufficient. In other words, just because something is not sufficient doesn't automatically mean it's necessary.

I hope this helps!

Jeremy
 theamazingrace
  • Posts: 59
  • Joined: Oct 17, 2020
|
#83546
Hi,

Was stuck between A and E. I considered A because it seemed true based on what the stimulus was saying just not as firm in the language in comparison to E. What is the real reason why is A is wrong?

Thank you in advance!
 Adam Tyson
PowerScore Staff
  • PowerScore Staff
  • Posts: 5153
  • Joined: Apr 14, 2011
|
#84438
Answer A is far too strong and broad in its scope. We cannot say that protection rarely enables this kind of growth, because we can only say that it didn't, and will not do so, in this country. This country may represent the rare case, rather than being the usual result!
User avatar
 Azimat
  • Posts: 4
  • Joined: Apr 06, 2021
|
#91711
I selected E as well but what makes B wrong? Is it because they call it an "economic policy"?
 Adam Tyson
PowerScore Staff
  • PowerScore Staff
  • Posts: 5153
  • Joined: Apr 14, 2011
|
#91731
Not exactly, Azimat, because this argument is about an economic policy. The problem with answer B is that it is too broad. We don't know what is true of ANY economic policy, but only about what is true of THIS economic policy! Answer B goes too far, and we don't have any evidence in the passage to show that 10 years is enough for any such policy. Some policies might take much longer to assess accurately, and that would not conflict with the facts in the passage.

Get the most out of your LSAT Prep Plus subscription.

Analyze and track your performance with our Testing and Analytics Package.